50 Points! Multiple choice geometry question. Photo attached. Thank you!

50 Points! Multiple Choice Geometry Question. Photo Attached. Thank You!

Answers

Answer 1

Answer:

The 13.2 meters from that point on the ocean floor to the wreck.

Step-by-step explanation:

As given

A ship's sonar finds that the angle of

depression to a ship wrack on the bottom of the ocean is 12.5°.

If a point on the ocean floor is 60

meters.

Now by using the trigonometric identity.

tano = Perpendicular /Base

As shown in the figure given below.

Perpendicular CB

Base AC = 60 meters

0 = 12.5°

Put in the identity CB

tan 12.5°

AC

tan 12.5° CB 60

tan 12.5° 0.22

tan 12.5° = 0.22

0.22 CB 60

CB = 60 × 0.22

CB= 13.2 meters

Therefore the 13.2 meters from that point on the ocean floor to the wreck.


Related Questions

sam says a roof is 12metres long and 7metres wide enough to install 6 solar panels verify whether his statement is correct by showing working out for the length of the roof using the width of the panel(1metres)

Answers

Since 6 is less than 84, the statement is correct. The roof with dimensions 12 meters by 7 meters is sufficient to accommodate 6 solar panels, each with a width of 1 meter.

To verify Sam's statement, we can calculate whether a roof that is 12 meters long and 7 meters wide can accommodate 6 solar panels, with each panel having a width of 1 meter.

The total area required to install 6 solar panels with a width of 1 meter each is:

6 panels * 1 meter = 6 square meters

Now, let's calculate the area of the roof:

Area of the roof = length * width = 12 meters * 7 meters = 84 square meters

Since the area required for the solar panels is 6 square meters and the area of the roof is 84 square meters, we can determine if the statement is correct.

If 6 square meters is less than or equal to 84 square meters, then the roof is indeed enough to install the 6 solar panels.

6 square meters ≤ 84 square meters.

For such more questions on Accommodate:

https://brainly.com/question/24462940

#SPJ11

S
9. Is the square root of 5.7 a rational number?

Answers

Answer:

Not rational

Step-by-step explanation:

A rational number is any integer, fraction, terminating decimal, or repeating decimal.

Anything with a square root is automatically not a rational number.

have a great day and thx for your inquiry :)

8. Choose the correct answer.

Determine the roots of the quadratic function shown in the graph.

SEE IMAGE TO SOLVE PROBLEM

Answers

Answer:

This quadratic function has no roots.

There are no roots of the quadratic function shown in the graph

We we consider the quadratic equation given be of the form ax^2 + bx + c = 0, then

The quantity b^2 - 4ac is known its discriminant.

The solution contains the term [tex]\sqrt{b^2 - 4ac}[/tex] which will be:

Real and distinct if the discriminant is positive

Real and equal if the discriminant is 0

Non-real and distinct roots if the discriminant is negative

Therefore, it has to be two roots of a quadratic equation always(assuming existence of complex numbers). We say that the considered quadratic equation has 2 solutions if roots are distinct, and have 1 solution when both roots are the same.

Hence, we can conclude that there are no roots. so option D is correct.

Learn more about quadratic functions here:

https://brainly.com/question/23100817

#SPJ1

During a summer reading program, Mary read 9 books. The books contained 217 pages, 138 pages, 159 pages, 356 pages, 270 pages, 112 pages, 138 pages, 210 pages, and 195 pages. What was the median number of pages of the 9 books that Mary read during the summer reading program?

270 pages

159 pages

202.5 pages

195 pages

Answers

The median number of pages of the 9 books that Mary read during the summer reading program is  195 pages .

To find the median number of pages of the 9 books that Mary read during the summer reading program, we first need to arrange the pages in ascending order:

112 pages, 138 pages, 138 pages, 159 pages, 195 pages, 210 pages, 217 pages, 270 pages, 356 pages

The median is the middle value in this ordered list. Since there are an odd number of values, the middle value is the fifth one, which is 195 pages.

The median is a measure of central tendency that divides a dataset into two equal halves. In this case, it represents the number of pages that is halfway between the smallest and largest values. It is a useful measure because it is less affected by outliers or extreme values than the mean.

In Mary's case, the median number of pages of the 9 books she read during the summer reading program is 195 pages. This means that half of the books she read had fewer than 195 pages and half had more than 195 pages. It gives us a good idea of what to expect in terms of the length of the books she read.

Know more about  median   here:

https://brainly.com/question/14532771

#SPJ11

Evaluate the determinant:
3 2 -2
1 2 4
-1 3 2

Answers

The determinant of the given matrix is -43.

We are given that;

The matrix= 3 2 -2

1 2 4

-1 3 2

Now,

To evaluate the determinant of the matrix

[3  2 -2]

[1  2  4]

[-1 3  2]

you can use the rule of Sarrus. First, you need to copy the first two columns of the matrix and place them next to the third column:

[3  2 -2 | 3  2]

[1  2  4 | 1  2]

[-1 3  2 | -1 3]

Then, you can calculate the sum of the products of the three diagonals going from left to right:

(3 * 2 * 2) + (2 * 4 * -1) + (-2 * 1 * 3) = 12 -8 -6 = -2

and subtract from it the sum of the products of the three diagonals going from right to left:

(-1 * 2 * -1) + (3 * 4 * 3) + (2 * 1 * 2) = 1 +36 +4 =41

-2 -41 = -43.

Therefore, by the matrices the answer will be -43.

Learn more about matrices here:

https://brainly.com/question/13430728

#SPJ1

3. Do these patterns have a constant difference or a constant ratio or neither?
a.1,4,10,19
b.2,5,7,11
c.3,7,13,21
d.12,10,6,0
e.2,6,13,23
f.7,13,25,43​

Answers

patterns a and d have a constant difference, pattern c has a constant difference, and patterns b, e, and f do not have a constant difference or a constant ratio.

a. The pattern has a constant difference. The first term is 1, the second term is 4, which is 3 more than 1, the third term is 10, which is 6 more than 4, and the fourth term is 19, which is 9 more than 10.

b. The pattern does not have a constant difference or a constant ratio. The differences between consecutive terms are 3, 2, and 4, respectively, which are not constant. The ratios between consecutive terms are 2.5/2, 1.4/2.5, and 11/1.4, respectively, which are also not constant.

c. The pattern has a constant difference. The first term is 3, the second term is 7, which is 4 more than 3, the third term is 13, which is 6 more than 7, and the fourth term is 21, which is 8 more than 13.

d. The pattern has a constant difference. The first term is 12, the second term is 10, which is 2 less than 12, the third term is 6, which is 4 less than 10, and the fourth term is 0, which is 6 less than 6.

e. The pattern does not have a constant difference or a constant ratio. The differences between consecutive terms are 4, 7, and 10, respectively, which are not constant. The ratios between consecutive terms are 2.5/2, 13/2.5, and 23/13, respectively, which are also not constant.

f. The pattern does not have a constant difference or a constant ratio. The differences between consecutive terms are 6, 12, and 18, respectively, which are not constant.

The ratios between consecutive terms are 13/7, 25/13, and 43/25, respectively, which are also not constant.

To learn more on Ratios click:

brainly.com/question/13419413

#SPJ1

find the missing side length to the nearest tenth

Answers

Answer:

C) 7.8

Step-by-step explanation:

Use Pythagorean Theorem to find the missing hypotenuse x:

[tex]x=\sqrt{5^2+6^2}=\sqrt{25+36} =\sqrt{61} =7.8[/tex]

The matching choice is C.

what is the length of the midsegment of a trapezoid with bases 12 andb28?

the length of the midsegment a trapezoid with the given bases is.

Answers

Answer:

midsegment = 20

Step-by-step explanation:

the midsegment is half the sum of the bases, that is

midsegment = [tex]\frac{12+28}{2}[/tex] = [tex]\frac{40}{2}[/tex] = 20

In circle H with m/GHJ = 30 and GH = 5 units, find the length of are GJ. Round to the nearest hundredth. H J G​

Answers

Answer:

GJ ≈ 31.42 units

Step-by-step explanation:

the arc length GJ is calculated as

GJ = circumference of circle × fraction of circle

     = 2πr × [tex]\frac{30}{180}[/tex] ( r is the radius )

the radius HJ = 30 , then

GJ = 2π × 30 × [tex]\frac{1}{6}[/tex]

     = 60π × [tex]\frac{1}{6}[/tex]

     = 10π

     ≈ 31.42 units ( to the nearest hundredth )

Somebody PLEASE help me!
!! I will give brainlist !!

Determine the measure of each arc.

Answers

The measure of arc MN is 144 degrees

measure of POQ is 322 degrees

The measure of arc PQ  is 132 degrees

The measure of arc MN is 72×2 = 144 degrees

The measure of arc NQR is 180 degrees

We have to find measure of NQ

First we have to measure of POQ

92+22+POQ=180

POQ=180-114

POQ=66 degrees

Now measure of arc POQ is (95+66)×2 which is 322 degrees

Measure of arc QR is 72×2 = 114 degrees

The measure of MOR is 108×2 =216 degrees

The measure of PQ is 66×2 which is 132 degrees

To learn more on Coordinate Geometry click:

brainly.com/question/27326241

#SPJ1

i need help cause i don’t know

Answers

Using a pattern we can find the missing values in the table, these are:

A = 56B = 70C = 84

How to find the missing values in the table?

When you look at the table, you can see that for each increase of 1 unit in the value of x, we have an increase of 14 units in the value of y. So we have a really simple pattern:

f(x) = f(x - 1) + 14

Then to get the next values in the table, just add 14 to the previous ones.

A = 42 + 14 = 56

B = 56 + 14 = 70

C = 70 + 14 = 84

Thes are the 3 missing values in the table.

Learn more about tables at:

https://brainly.com/question/12151322

#SPJ1

Question: Margo Xavier makes a deposit at
an ATM. She has a paycheck for $375.42 and
a refund check for \$24.95. She would like to
receive $45.00 in cash and deposit the
remaining amount. What is her total deposit?

Answers

Hello !

total = $375,42 + $24,95 = $400,37

in cash = $45

the remaining amount = $400,37 - $45 = $355,37

There are six girls and ten boys in a class. Three of the girls and four of the boys wear glasses.
a The teacher chooses one person at random. What is the probability that the teacher chooses:

Answers

The probability that the teacher chooses a girl with glasses is 3/16, and the probability that the teacher chooses a boy with glasses is 1/4.

Let's calculate the probability that the teacher chooses a student with glasses.
a) A girl with glasses: There are 6 girls in the class, and 3 of them wear glasses. There are a total of 16 students (6 girls + 10 boys).
Probability = (Number of girls with glasses) / (Total number of students)
Probability = 3 girls with glasses / 16 total students
Probability = 3/16
b) A boy with glasses: There are 10 boys in the class, and 4 of them wear glasses.
Probability = (Number of boys with glasses) / (Total number of students)
Probability = 4 boys with glasses / 16 total students
Probability = 1/4
So, the probability that the teacher chooses a girl with glasses is 3/16, and the probability that the teacher chooses a boy with glasses is 1/4.

For more such questions on probability , Visit:

https://brainly.com/question/24756209

#SPJ11

4) What is <8 if <2 is 27°​

Answers

If Angle 2 Is-congruent-to Angle 6, then the statement which must be true about Angle 2 is that Angle 2 is supplementary to Angle 8.

We, have,

This refers to the type of angle where two different angles are said to have equal measure on their corresponding sides.

With this in mind, we can see that based on the two horizontal parallel lines are intersected by a third line, there is a third line that intersects the top line and forms four angles.

Hence, if If Angle 2 Is-congruent-to Angle 6, then the statement which must be true about Angle 2 is that Angle 2 is supplementary to Angle 8 because the sum of either angles is 180 degrees.

Read more about congruent angles here:

brainly.com/question/1675117

#SPJ1

complete queston:

Two lines are intersected by a third line.

Two horizontal parallel lines are intersected by a third line. The third line intersects the top line and forms 4 angles. Labeled clockwise, from uppercase left, the angles are 1, 2, 4, 3. The third line intersects the bottom line and forms 4 angles. Labeled clockwise, from uppercase left, the angles are 5, 6, 8, 7.

If Angle2 Is-congruent-to Angle6, which must be true about Angle2?

Angle2 Is-congruent-to Angle5

Angle2 is complementary to Angle5.

mAngle2 = mAngle8

Angle2 is supplementary to Angle8.

Us the de Morgan laws to write an equivalent statement using parenthesis.
P and not q.

Answers

The equivalent statement using parenthesis using the de Morgan laws would be (not q) or (not p).

De Morgan's Laws. To write an equivalent statement using parentheses for P and not Q (P ∧ ¬Q), we'll apply De Morgan's Law:
De Morgan's Law states that:
1. ¬(P ∧ Q) ≡ ¬P ∨ ¬Q
2. ¬(P ∨ Q) ≡ ¬P ∧ ¬Q
In your case, you have P ∧ ¬Q, which doesn't directly match either of these forms. However, we can manipulate it to apply De Morgan's Law by taking the negation of both sides and then applying De Morgan's Law:
Original statement: P ∧ ¬Q
Negate both sides: ¬(P ∧ ¬Q)
Now, we can apply De Morgan's Law to the negated statement:
¬(P ∧ ¬Q) ≡ ¬P ∨ ¬¬Q
Since ¬¬Q is equivalent to Q, we have:
¬(P ∧ ¬Q) ≡ ¬P ∨ Q
Now we have an equivalent statement with parentheses:
Your answer: ¬(P ∧ ¬Q) ≡ ¬P ∨ Q

For more such questions on parenthesis , Visit:

https://brainly.com/question/30250580

#SPJ11

The school cafeteria is thinking of introducing a vegetarian pizza to the menu. Which
sampling technique would you recommend to determine which of three possible
toppings should be used? Explain your recommendation. [C -2]
4.

Answers

I recommend use of simple random sampling to determine the three possible toppings to be used for the vegetarian pizza.

What is simple random sampling suitable?

It is a sampling technique in which each member of the population has an equal chance of being selected for the sample. Here, the population would be students who regularly eat at the school cafeteria.

By using the sampling, we make sure that each student has an equal chance of being selected to taste test the different pizza toppings. This will help to eliminate bias and provide a representative sample of the population's preferences. They will provide feedback which will be used to determine which topping should be used for the vegetarian pizza.

Read more about sampling technique

brainly.com/question/28292794

#SPJ1

write the standard equation and sketch a graph of the parabola with a Focus at (-2,1) and a directrix of y=8. Make sure to label all relevant features in your sketch

Answers

Answer: The standard equation of a parabola is given by:

4p(x - h) = (y - k)^2

where (h, k) is the vertex of the parabola, p is the distance from the vertex to the focus and also the distance from the vertex to the directrix.

In this case, the focus is at (-2,1), so the vertex is halfway between the focus and the directrix, which is at (x, y) = (-2, (1+8)/2) = (-2, 4.5). The distance from the vertex to the focus (and to the directrix) is p = 3.5.

Substituting these values into the equation, we get:

4(3.5)(x + 2) = (y - 4.5)^2

Simplifying, we get:

14(x + 2) = (y - 4.5)^2

This is the standard equation of the parabola.

To sketch the graph, we can use the vertex (-2, 4.5) as a starting point, and use the distance from the vertex to the focus to plot some points on either side of the vertex. Since the focus is to the left of the vertex, we know that the parabola will open to the left.

Using the distance formula, we can find the coordinates of two points on the parabola that are equidistant from the focus and the directrix:

Point 1: (-5.5, 4.5)

Distance from focus: sqrt((-5.5 + 2)^2 + (4.5 - 1)^2) = sqrt(44.5) ≈ 6.67

Distance from directrix: |4.5 - 8| = 3.5

Point 2: (-5.5, 1.5)

Distance from focus: sqrt((-5.5 + 2)^2 + (1.5 - 1)^2) = sqrt(36.5) ≈ 6.04

Distance from directrix: |1.5 - 8| = 6.5

Parabola with a focus at (-2,1) and directrix y=8

The vertex is labeled as V, the focus as F, and the directrix as D. The distance from the vertex to the focus (and directrix) is labeled as p. The parabola opens to the left and is symmetric about the axis of symmetry, which is the vertical line passing through the vertex.

For the equation:
to have an infinite number of solutions, what must the value of a be?
Elimination Tool
Select one answer
A 5
B 10
C 15
D 20
1
5(z 3) + a=5(z + 1)
25

Answers

To have an infinite number of solutions the value of a would be

D. 20

How to find the value of a

Information given in the problem includes

5(x - 3) + a = 5(x + 1)

To have an infinite number of solutions we can simplify as follows

5(x - 3) + a = 5(x + 1)

opening the parenthesis

5x - 15 + a = 5x + 5

collecting like terms

-15 + a = 5

Adding 15 to both sides, we get:

a = 20

Learn more about infinite number of solution at

https://brainly.com/question/27927692

#SPJ1

If 12 apples are used for every 7 pounds of dough, how many apples are needed if we have 115 pounds of dough?

Answers

Hello !

Answer:

198 apples

Step-by-step explanation:

12 apples <=> 7 pounds of dough

x apples <=> 115 pounds of dough

Let's use a cross product :

[tex]x = \frac{12 \times 115}{7} \approx197.14 [/tex]

198 apples are needed.

Have a nice day

Solve for the missing side length. Round to 2 decimal places
23
63°
X
Type a response

Answers

The value of missing side length is, 51.11

We have to given that;

A triangle is shown.

And, To find the missing side of triangle.

Since, An angle is a combination of two rays (half-lines) with a common endpoint. The latter is known as the vertex of the angle and the rays as the sides, sometimes as the legs and sometimes the arms of the angle.

And, A triangle is a three sided polygon, which has three vertices and three angles which has the sum 180 degrees.

Now, By using trigonometry formula, we get;

⇒ cos 63° = Hypotenuse / Base

⇒ cos 63° = 23 / x

⇒ 0.45 = 23 / x

⇒ x = 23 / 0.45

⇒ x = 51.11

Therefore, The value of missing side in the triangle is,

⇒ x = 51.11

Learn more about the triangle visit;

brainly.com/question/1058720

#SPJ1

The Earth's human population is approximately 7,900,000,000 people.
Express this number using scientific notation.

Answers

The Earth's human population, approximately 7,900,000,000 people, can be expressed using scientific notation as 7.9 x 10^9.

Scientific notation is used to express extremely large numbers using a number between 1 and 10.

In this case, that number is 7.9

We need to move that decimal spot 9 places to the right to get 7,900,000,000. Therfore, to state that number in scientific notation, we would write it as:

7.9 * 10^9

I hope this helped!

~~~Harsha~~~

Find the exact value of sin75°cos15° - cos75°sin15°

Answers

Answer:

√3/2

Step-by-step explanation:

Easy Method

The equation above is in the forms of sin(a)cos(b) - cos(a)sin(b), which is sin(a-b) according to the trig identities. sin(75-15) = sin(60) = √3/2

Harder Method

Find sin75 with equation sin(a+b) = sin(a)cos(b) + cos(a)sin(b)

sin75°

= sin(45° + 30°)

= [sin45°cos30° + cos45°sin30°]

= [√2/2 * √3/2 + √2/2 * 1/2]  <-- unit circle known values

= [(√6 + √2)/4]

Find cos75 with the equation: cos(a+b) = sin(a)sin(b) - cos(a)cos(b)

cos75°

= cos(45° + 30°)

= [cos45°cos30° - sin45°sin30°]

= [√2/2 * √3/2 - √2/2 * 1/2]  <-- unit circle known values

= [(√6 - √2)/4]

Find sin15 with the equation: sin(a-b) = sin(a)cos(b) - cos(a)sin(b)

sin15°

= sin(45° - 30°)

= [sin45°cos30° - cos45°sin30°]

= [√2/2 * √3/2 - √2/2 * 1/2]  <-- unit circle known values

= [(√6 - √2)/4]

Find cos15 with the equation: cos(a-b) = sin(a)sin(b) + cos(a)cos(b)

cos15°

= cos(45° - 30°)

= [cos45°cos30° + sin45°sin30°]

= [√2/2 * √3/2 + √2/2 * 1/2]  <-- unit circle known values

= [(√6 + √2)/4]

Now plug in all the solved values, we get: {[(√6 + √2)/4] * [(√6 + √2)/4]} - {[(√6 - √2)/4] * [(√6 - √2)/4]} = √3/2

Please friends I need help

Answers

The solution to the system of equations, using the Gauss-Jordan method, is given as follows:

[tex]\left[\begin{array}{cccc}1&0&0&14.28\\0&1&0&-21.4\\0&0&1&1.2\end{array}\right][/tex]

How to solve the system of equations?

The matrix representing the system of equations is given as follows:

[tex]\left[\begin{array}{cccc}-5&-3&-4&-12\\0&-2&-7&38\\0&1&4&-22\end{array}\right][/tex]

First we want a value of 1 at line 1, column 1, hence we multiply the first line by -1/5, that is:

R1 -> -1/5R1

Hence:

[tex]\left[\begin{array}{cccc}1&0.6&0.8&2.4\\0&-2&-7&38\\0&1&4&-22\end{array}\right][/tex]

First we want a value of 1 at line 2, column 2, hence we multiply the second line by -1/2, that is:

R2 -> -1/2R2

Hence:

[tex]\left[\begin{array}{cccc}1&0.6&0.8&2.4\\0&1&1.5&-19\\0&1&4&-22\end{array}\right][/tex]

We want an element of zero at line 3, column 2, hence:

R3 -> R3 - R2

Hence:

[tex]\left[\begin{array}{cccc}1&0.6&0.8&2.4\\0&1&1.5&-19\\0&0&2.5&-3\end{array}\right][/tex]

First we want a value of 1 at line 3, column 3, hence we multiply the third line by 2.5, that is:

R3 -> 1/2.5R3

Hence:

[tex]\left[\begin{array}{cccc}1&0.6&0.8&2.4\\0&1&1.5&-19\\0&0&1&1.2\end{array}\right][/tex]

Then the solution to the system of equations is given as follows:

z = 1.2.y = -19 - 1.5z = -21.4.x = 2.4 - 0.6y - 0.8z = 14.28.

Hence the row-echelon form is given as follows:

[tex]\left[\begin{array}{cccc}1&0&0&14.28\\0&1&0&-21.4\\0&0&1&1.2\end{array}\right][/tex]

More can be learned about the row echelon form of matrices at https://brainly.com/question/28968080

#SPJ1

what is the equation of the function represented by the table of values

Answers

y=3(5)ˣ is the equation of the function represented by the table

We can see that as x increases, y increases exponentially.

To determine the specific form of the exponential function, we can take the ratio of successive y-values:

(3/5)/(3/25) = 5/1

(3)/(3/5) = 5

(15)/(3) = 5

(75)/(15) = 5

This shows that the ratio of successive y-values is constant at 5. Therefore, the function represented by the table is an exponential function of the form:

y = a(b)ˣ

To find a and b, we can use any two pairs of (x,y) values.

Let's use (0,3) and (1,15):

3 = a(b)⁰

15 = a(b)¹

3 = a

Now we get b =5

Hence, y=3(5)ˣ is the equation of the function represented by the table

To learn more on Functions click:

https://brainly.com/question/30721594

#SPJ1

A 40 ft ladder is leaning against a wall making a 48° angle with the ground.
Draw a diagram that you can use to determine approximately how far the base of the
ladder is from the wall.

Answers

Using a trigonometric relation we can see that the distance between the base and the wall is 26.76ft

How to find the distance between the base and the wall?

In the image at the end you can see a diagram for this problem.

We have a right triangle where we want to find the value of d, which is the adjacent cathetus to the known angle.

Then we can use the trigonometric relation:

cos(a) = (adjacent cathetus)/hypotenuse

Where:

a = 48°

hypotenuse = 40ft

adjacent cathetus = d

Replacing that we will get:

cos(48°) = d/40ft

Solving that for d, we will get:

40ft*cos(48°) =d

26.76ft = d

That is the distance between the base and the wall.

Learn more about right triangles at:

https://brainly.com/question/2217700

#SPJ1

Paul is playing a board game and rolls two number cubes. Let A = (the sum of the number cubes is odd), and let B = (the sum of the number cubes is divisible by 4). List the outcomes in A U B.

Answers

The outcomes in set A U B are 1, 3, 5, 6, 7, 8, 9, 10, 11, 12 (the odd sums) 4, 8, 12 (the sums divisible by 4)

The sum of the number cubes is odd if one die shows an even number and the other shows an odd number, or vice versa.

There are 3 even numbers and 3 odd numbers on a number cube, so there are 3 x 3 = 9 ways to get an odd sum.

The sum of the number cubes is divisible by 4 if both dice show either 2 or 4.

There are 2 ways to get a 2 on a number cube and 2 ways to get a 4, so there are 2 x 2 = 4 ways to get a sum of 4.

Therefore, the outcomes in A U B are 1, 3, 5, 6, 7, 8, 9, 10, 11, 12 (the odd sums) 4, 8, 12 (the sums divisible by 4)

To learn more on Sets click:

https://brainly.com/question/8053622

#SPJ1

Determine whether the correlation coefficients show strong, moderate, or weak correlation.
Strong Correlation
Moderate Correlation
r=-0.91
r=0.82
r=-049
r=026
r=0.54
r=-0.18
Intro
Weak Correlation
Done

Answers

Answer:

-0.91 = strong, as it is close to -1 and if it's close to -1 that means it is linear.

0.82 = strong. Again, it's approaching 1 and is, therefore, approaching linearity.

-0.49 = moderate

0.26= weak

0.54 = moderate

-0.18 = weak

Step-by-step explanation:

can someone help me with this?

Answers

In the given diagram, the area of the shape is 2π units²

Calculating the area of a Quadrant

From the question, we are to calculate the area of the given shape.

In the given diagram, the shape is a quadrant.

The area of a quadrant is given by the formula

Area = 1/4 πr²

Where r is the radius

From the given information,

Radius = 8

Thus,

Area of the quadrant = 1/4 × π × 8

Area of the quadrant = 2π units²

OR

Area of the quadrant = 2 × 3.14 units²

Area of the quadrant = 6.28 units²

Hence,

The area of the shape is 2π units² OR 6.28 units².

Learn more on Calculating the area of a Quadrant here:

https://brainly.com/question/16501078

#SPJ1

If f(x) = 3 + 3, which of the following is the inverse of f(x)?
O A. f-¹(x) = 3(x+3)
5
OB. f¹(x) =
3(x-3)
5
O c. f-¹(x) =
5(z-3)
○ D. f−1(x) = 5(x+3)

Answers

The inverse of f(x) is f⁻¹(x) = (x - 3)/3.

To find the inverse of the function f(x), we need to switch the roles of x and y and solve for y.

Given:

f(x) = 3x + 3

Step 1: Replace f(x) with y:

y = 3x + 3

Step 2: Swap x and y:

x = 3y + 3

Step 3: Solve for y:

x - 3 = 3y

3y = x - 3

y = (x - 3)/3

Therefore, the inverse of f(x) is f⁻¹(x) = (x - 3)/3.

Learn more about Inverse of Function here:

https://brainly.com/question/2541698

#SPJ1

Hi help please quick

Answers

Answer:

Initial amount: 934 g

After 80 years: 147 g

Step-by-step explanation:

[tex] A(t) = 934 (\dfrac{1}{2})^\frac{t}{30} [/tex]

The initial amount occurs at t = 0.

[tex] A(0) = 934 (\dfrac{1}{2})^\frac{0}{30} [/tex]

[tex] A(0) = 934 (\dfrac{1}{2})^0 [/tex]

[tex] A(0) = 934 \times 1 [/tex]

[tex] A(0) = 934 [/tex]

After 80 years, t = 80.

[tex] A(80) = 934 (\dfrac{1}{2})^\frac{80}{30} [/tex]

[tex] A(80) = 934 (\dfrac{1}{2})^\frac{8}{3} [/tex]

[tex] A(80) = 934 (0.15749) [/tex]

[tex] A(80) = 147 [/tex]

Other Questions
how would a concerto soloist let the conductor know that a cadenza was ending? francine currently has $30,000 in her 401k account at work, and plans to contribute $3,000 each year for the next 10 years. how much will she have in the account in 10 years, if the account averages a 4% annual return? suppose that the chinese government has a program that pays $1000 in transfer to each newborn chinese baby outside of the country. in 2020, there is a chinese overseas baby boom: the number of newborn babies increases 10 times. suppose the chinese government is running a deficit, what will happen to chinese government debt at that time? what are two changes that will need to be made to the budget in later years if china is not going to default? a factory has two machines A and B producing 300 and 720 bulbs per day respectively. machine A produces 1% defective bulbs and machine B produces 1.5% defective bulbs. One bulb is chosen at random at the end of a day and found. what is the probability that machine B produce defective bulb ? 4. What superseded the threat of nuclear conflict following the collapse of the Soviet Union? Can someone answer this for me ( field of view lab) agencia de viajes quiere incrementar el turismo nacional por carretera, para ello genera una estrategiapublicitaria, cuyos resultados exitosos se veran reflejados cuandoA. se mantengan los porcentajes de respuesta a la pregunta 2B. se aumente el porcentaje de personas que prefieren viajar a lugares cercanos a su residencia, en lapregunta 3C. los porcentajes de respuesta a la pregunta 1 quedan intercambiados y se mantengan los porcentajes enlas otras preguntasD. se disminuyan los porcentajes de los que no prefieren viajar por carretera, en la pregunta Amit can complete a piece of work in 2.25 days. Badri takes double the time taken by Amit. Chetantakes double that of Badri, and Das takes double that of Chetan to complete the same task. They aresplit into two groups (of one or more persons) such that the difference between the times taken bythe two groups to complete the same work is minimum. What could be the composition of the fastergroup? All of the following statements about the Great Steel Strike of 1919 are true EXCEPT:A) the strike involved mostly nonimmigrant workers.B) the strike centered in Chicago.C) the strike involved 365,000 workers.D) workers demanded union recognition.E) workers won an eight-hour day. intervening sequences that are removed from a mrna before it leaves the nucleus are called: how would the salvage value be treated in a net present value calculation?multiple choicedisregard the salvageas a positive cash flow in the final year that the asset is usedas a negative cash flow in the final year that the asset is usedas a negative cash flow in the first year that the asset is used sports teams, labor unions, and a company's employees are examples of _____. a stock is currently selling for $46. over the next two periods, the stock will move up by a factor of 1.35 or down by a factor of .89 each period. a call option with a strike price of $62 is available. if the risk-free rate of interest is 3.2 percent per period, what is the value of the call option? (do not round intermediate calculations. round your answer to 2 decimal places.) what is the first task in most military air campaigns? The satrapies subject to the authority of Darius:a. were delegated little authorityb. enjoyed considerable independencec. administered independently except in matters of foreign policyd. were not permitted to collect revenuee. administered independently except in matters of justice Does a change in the molality cause the boiling point of a solution to increase or decrease?. the beginning stages of the masters and johnson technique of sensate focus involve _______. which component of a presentation is analogous to the conclusion in a written document? Which of the following is the correct sequence of steps in the repair of a bone fracture?a) Fracture hematoma, bony callus formation, fibrocartilaginous callus formation, bone remodelingb) Fibrocartilaginous callus formation, bony callus formation, fracture hematoma, bone remodelingc) Bony callus formation, fracture hematoma, fibrocartilaginous callus formation, bone remodelingd) Fracture hematoma, fibrocartilaginous callus formation, bony callus formation, bone remodeling When the spleen enlarges, the nurse would not be surprised to percuss dullness over the stomach.a) Trueb) False